cardiology incorrects Flashcards

1
Q

50 YO, atrial fibrillation picked up incidentally, pulse rate 80bpm, chadsvasc score is 0

what is the next step in management?

A

no medication!!

patient has non symptomatic AF. rate is controlled. no other RFs

How well did you know this?
1
Not at all
2
3
4
5
Perfectly
2
Q

A 63 year old woman has episodes of irregular palpitations, lasting several days and occurring once a month. She has ischaemic heart disease and type 2 diabetes.Her ECG confirms atrial fibrillation.
The patient wants to discuss the risks before starting anticoagulation (see image).
Which is her lifetime risk of having a stroke related to her atrial fibrillation?
A. 3%
B. 15%
C. 20%
D. 30%
E. 60%

UKMLA ppq!

A

Correct Answer(s): E
Justification for correct answer(s): 60% as the her CHADsVasc score is 3

From the table her adjusted stroke risk is 3.2

so 3% per year with life expectancy from 63 years to be about 20 years (83) (2020: life expectancy is 82 years). 3 x 20 is 60%. When discussing this with patients it puts it into perspective more if its lifetime rather than annual risk. Patients more likely to choose anticoagulation.

How well did you know this?
1
Not at all
2
3
4
5
Perfectly
3
Q

A 88 year old woman has 30 hours of severe abdominal pain. She was discharged from hospital 4 weeks ago following an aortobifemoral bypass graft.She has diffuse tenderness of the abdomen with absent bowel sounds. Plain X-ray of the abdomen is unremarkable.Investigations:Haemoglobin 114 g/L (115-160)
White cell count 18 × 109/L (3.8–10.0)Urea 16.4 mmol/L (2.5–7.8) Creatinine 158 μmol/L (60–120)CRP 110 mg/L (<5)Arterial blood gas breathing air
pH 7.28 (7.35–7.45)
PO2 13 kPa (11–15)
PCO2 4.6 kPa (4.6–6.4)
Bicarbonate 16 mmol/L(22–30)Lactate 4.5 mmol/L (1–2)

UKMLA ppq

A

Mesenteric ischemia
History of vascular disease and lactic acidosis

How well did you know this?
1
Not at all
2
3
4
5
Perfectly
4
Q

side effects of amiodarone?

A

thyroid dysfunction: both hypothyroidism and hyper-thyroidism
corneal deposits
pulmonary fibrosis/pneumonitis
liver fibrosis/hepatitis
peripheral neuropathy, myopathy
photosensitivity
‘slate-grey’ appearance
thrombophlebitis!! and injection site reactions
bradycardia
lengths QT interval

How well did you know this?
1
Not at all
2
3
4
5
Perfectly
5
Q

inhaled foreign objects are most likely found where?

A

Right inferior lobe bronchus

How well did you know this?
1
Not at all
2
3
4
5
Perfectly
6
Q

a diagnosis of arrhythmogenic right ventricular cardiomyopathy (ARVC).

What abnormality is most likely to have been present on the patient’s ECG?

A

T-wave inversion in V1-3
40%

How well did you know this?
1
Not at all
2
3
4
5
Perfectly
7
Q

A 35-year-old male presents with pain in both legs for the last two weeks. He has noted that the pain occurs after he has walked for ten minutes during his morning walk and is relieved when he sits for some time. There is no swelling but has noted that his toes turn white, then blue and red during the cold. There is no history of trauma. He does not drink alcohol but has been smoking three to four packs of cigarettes per day for ten years.

Which of the following is the most likely diagnosis?

A

Thromboangiitis obliterans/ buergers disease

How well did you know this?
1
Not at all
2
3
4
5
Perfectly
8
Q

A 63-year-old man with a history of hypertension and stable angina is currently on 20mg bisoprolol OD, aspirin 75mg OD, atorvastatin 80 mg OD, and glyceryl trinitrate (GTN) spray PRN. He remains compliant with his medication; however, he continues to experience chest pain during moderate physical activity. His heart rate is 80 bpm, and his blood pressure is 120/80 mmHg.

Which medication would be most appropriate to add to this man’s regimen?

A

amlodipine

If angina is not controlled with a beta-blocker, a longer-acting dihydropyridine calcium channel blocker should be added

How well did you know this?
1
Not at all
2
3
4
5
Perfectly
9
Q

colleague asks you to pass her the amiodarone syringe for infusion. You find that the crash trolley has been restocked incorrectly and the syringe is missing.

What drug can be substituted in this situation?

A

lidocaine

How well did you know this?
1
Not at all
2
3
4
5
Perfectly
10
Q

which valve condition is associated with polycystic kidney disease?

A

mitral valve prolapse

How well did you know this?
1
Not at all
2
3
4
5
Perfectly
11
Q

A 47-year-old man presents to the emergency department with sudden onset central chest pain. The patient has a past medical history of hypertension for which he is taking ramipril. His observations are BP 153/90mmHg in his right arm and 130/80 in his left arm, heart rate 89/minute, temperature 37.2ºC, respiration rate 17/minute, and oxygen saturation 98% on room air. On examination, pulses are absent in the right leg and diminished in the left.

diagnosis?
What is the preferred investigation for this patient’s presentation?

A

aortic dissection

CT angiography chest/abdomen/pelvis

How well did you know this?
1
Not at all
2
3
4
5
Perfectly
12
Q

Renal dysfunction (eGFR < 60) can cause a raised serum natriuretic peptides/ BNP

A
How well did you know this?
1
Not at all
2
3
4
5
Perfectly
13
Q

A 35-year-old woman presents to the ED with acute shortness of breath following a motor vehicle accident 30 minutes ago. On examination, her blood pressure is 82/45 mmHg, the pulse is 120 bpm, capillary refill time is 3 seconds, and JVP is 6 cm above the sternal angle. A 500 mL bolus of IV crystalloid was given, but she remains hypotensive and tachycardic.

Given the most likely cause of this presentation, what is the most appropriate management?

A

pericardiocentesis

cardiac tamponade

How well did you know this?
1
Not at all
2
3
4
5
Perfectly
14
Q

when should simvastatin be taken?

A

last in the evening

How well did you know this?
1
Not at all
2
3
4
5
Perfectly
15
Q

Mobitz type 1 (Wenckebach phenomenon) is a normal variant in an athlete - discharge with safety netting advice

A
How well did you know this?
1
Not at all
2
3
4
5
Perfectly
16
Q

major bleeding on warfarin management?

A

Give intravenous vitamin K 5mg and prothrombin complex concentrate

How well did you know this?
1
Not at all
2
3
4
5
Perfectly
17
Q

statins + what drug is a common interaction?

A

erythromycin/clarithromycin

How well did you know this?
1
Not at all
2
3
4
5
Perfectly
18
Q

You are working in a GP practice. Your next patient is a 27-year-old female who has just found out she is 6 weeks pregnant. She has a past medical history of familial hypercholesterolaemia, type 1 diabetes and asthma. What should your next step in management be?

A

stop statin -> contraindicated in pregnancy

How well did you know this?
1
Not at all
2
3
4
5
Perfectly
19
Q

Ivabradine use may be associated with visual disturbances including phosphenes and green luminescence

A
How well did you know this?
1
Not at all
2
3
4
5
Perfectly
20
Q

what are the warfarin inr targets in valve replacement?

A

Mechanical valves - target INR:
aortic: 3.0
mitral: 3.5

How well did you know this?
1
Not at all
2
3
4
5
Perfectly
21
Q

learn hypokalemia presentation on ecg, can be caused by furosemide

A
How well did you know this?
1
Not at all
2
3
4
5
Perfectly
22
Q

A 64-year-old man presents to the respiratory clinic for a scheduled review of his pulmonary hypertension. He was diagnosed with pulmonary hypertension ten years previously. His comorbidities include type 2 diabetes mellitus and heart failure. His medications include bosentan, metformin, sitagliptin, ramipril and bisoprolol.

On examination, he is slightly overweight; he has mild peripheral oedema; his lungs are clear; and there is a high-pitched, pansystolic murmur, heard loudest at the lower left sternal edge, which is loudest on inspiration.

Which underlying pathology is most likely to explain his murmur?

A

ttricuspid regurg

RILE

and then systolic murmur so tricuspid regurbg not stenosis

How well did you know this?
1
Not at all
2
3
4
5
Perfectly
23
Q

Lung crackles heard on auscultation/ cardiogenic shock is a poor prognostic indicator in acute coronary syndrome

A
How well did you know this?
1
Not at all
2
3
4
5
Perfectly
24
Q

common adverse effect of indapamide?/ thiazide like diuretics

A

erectile dysfunction

How well did you know this?
1
Not at all
2
3
4
5
Perfectly
25
Q

stable angina first line treatment?

A

A beta-blocker or a calcium channel blocker eg dilitiazem

How well did you know this?
1
Not at all
2
3
4
5
Perfectly
26
Q

beta blocker side effects?

A

bronchospasm
cold peripheries
fatigue
insomnia, including nightmares!!!
erectile dysfunction

How well did you know this?
1
Not at all
2
3
4
5
Perfectly
27
Q

A 46-year-old woman presents to the medical assessment unit with a four-month history of shortness of breath and lightheadedness on exertion. On examination, she has a systolic murmur. When palpating her radial artery, a double pulse is felt during systole.

What condition is most likely to produce this pulse?

A

mixed aortic valve disease

How well did you know this?
1
Not at all
2
3
4
5
Perfectly
28
Q

New LBBB is always pathological and never normal even in an athlete

A
How well did you know this?
1
Not at all
2
3
4
5
Perfectly
29
Q

antiplatelet regime following a stroke?

A

Aspirin 300 mg daily for 2 weeks then clopidogrel 75 mg daily lifelong

How well did you know this?
1
Not at all
2
3
4
5
Perfectly
30
Q

A 14-year-old boy presents to the Emergency Department as he is unable to control his facial muscles and arm movements. For the last 5 weeks, following a throat infection, he has been experiencing ongoing fever, worsening shortness of breath and joint pains, mainly in his legs which have not been effectively managed. What is the most likely cause of the patient’s recent symptoms?

A

Sydenham’s chorea - complication of rhuematic fever

How well did you know this?
1
Not at all
2
3
4
5
Perfectly
31
Q

warfarin interacts with fluconazole!!

A
How well did you know this?
1
Not at all
2
3
4
5
Perfectly
32
Q

INR 5.0-8.0 (no bleeding) - withhold 1 or 2 doses of warfarin, reduce subsequent maintenance dose

A
32
Q

premature ventricular beats -> effect on electrolyte balance, how it looks on ECG

A
33
Q

If new BP >= 180/120 mmHg + new-onset confusion, chest pain, signs of heart failure, or acute kidney injury then admit for specialist assessment

A
34
Q

classic causes of dilated cardiomyopathy?

A

alcohol!!
Coxsackie B virus
wet beri beri
doxorubicin

35
Q

what can be added as a second line treatment for rate control in atrial fibrillation?

A

digoxin

35
Q

Poorly controlled hypertension, already taking an ACE inhibitor and a thiazide diuretic - add a calcium channel blocker

dont add arb because already using ACE

A
36
Q

A 22-year-old woman presents to the clinic after being referred by her GP. She has a six-month history of unexplained malaise and headaches. She feels fatigued and finds walking for long distances extremely challenging as her legs start aching after she walks more than 100m.

On examination, her radial pulse is weaker on the right side. Her dorsalis pedis and posterior tibial pulses are absent bilaterally. An early diastolic decrescendo murmur heard is present. She is a non-smoker, and she is otherwise healthy.

Given the most likely diagnosis, what investigation is needed to confirm it?

A

characteristic features of Takayasu’s arteritis leading to absent peripheral pulses (as in this case) or uneven blood pressure and pulses (as shown in the radial pulse). Additionally, the occlusion can cause symptoms similar to claudication due to the reduced blood flow, described as aching legs. It has been associated with aortic regurgitation (an early diastolic decrescendo murmur) in 20% of the cases. Vascular imaging is required to confirm the diagnosis; in this case, magnetic resonance angiography (MRA) is the only viable option.

37
Q

or an SVT, the Resus Council recommend escalating adenosine doses of 6mg → 12mg → 18 mg

A
38
Q

A 65-year-old man attends a stroke clinic following a transient ischaemic attack.

On examination, he has a diastolic murmur loudest over the apex. His pulse rate is 90 bpm and irregular, blood pressure is 130/90 mmHg, and respiratory rate is 20 breaths per minute. An ECG shows an irregular ventricular rate and absent P waves.

What is the most likely cause of the murmur?

A

diastolic murmur + AF = mitral stenosis

39
Q

A 48-year-old man has heart failure. He attends the emergency department because he feels short of breath and has gained 11kg in weight. His blood pressure is 88/48 mmHg, heart rate 112 bpm, and is requiring 2L of oxygen. Capillary refill time is 3 seconds, and you can feel his liver edge 6cm below the costal margin. Regular medications include 40mg furosemide twice per day and ramipril.

His renal profile last month was normal. His U&Es on this admission are as follows:

Na+ 131 mmol/L (135 - 145)
K+ 4.0 mmol/L (3.5 - 5.0)
Bicarbonate 17 mmol/L (22 - 29)
Urea 14.7 mmol/L (2.0 - 7.0)
Creatinine 248 µmol/L (55 - 120)

What is the most appropriate treatment?

A

160mg IV furosemide infusion
Increased doses of loop diuretics may be required in patients with poor renal function to ensure sufficient concentration is achieved within the tubules

40
Q

baby, An echocardiogram was performed which confirmed a diagnosis of coarctation of the aorta.

What is the most appropriate next step in management after resuscitation?

A

Immediate administration of prostaglandin

41
Q

A 74-year-old woman is reviewed. She recently had ambulatory blood pressure monitoring that showed an average reading of 142/90 mmHg. There is no significant past medical history of note other than hypothyroidism. Her 10-year cardiovascular risk score is 23%. What is the most appropriate management?

A

Newly diagnosed patient with hypertension (> 55 years) - add a calcium channel blocker

42
Q

a fib pharmacological cardioversion drugs?

A

amiodarone + flecainide

43
Q

An 82-year-old man is reviewed. He is known to have ischaemic heart disease and is still getting regular attacks of angina despite taking atenolol 100mg od. Examination of his cardiovascular system is unremarkable with a pulse of 72 bpm and a blood pressure of 158/96 mmHg. What is the most appropriate next step in management?

A

If angina is not controlled with a beta-blocker, a longer-acting dihydropyridine calcium channel blocker should be added

44
Q

learn ST elevations and the arteries affected eg!!!
A 59-year-old female is admitted to the Emergency Department with a 30 minute history of central chest pain radiating to her left arm. An ECG shows ST elevation in leads II, III, aVF. Which coronary artery is most likely to be affected?

A

right coronary artery

45
Q

A 71-year-old man presents to the emergency department with an abnormal cardiac rhythm. After various unsuccessful attempts of restoring normal cardiac rhythm, the team decides to administer him a drug. After the administration, he complains of severe chest pain, which is self-limiting and terminates quickly.

What drug has this patient been given?

A

adenosine - may cause chest pain

46
Q

A 58-year-old man with predictable chest pain on exertion sees his GP to discuss medication. He has already undergone investigations and was diagnosed with angina. The GP starts him on aspirin and a statin.

What would be the most appropriate prophylactic medication?

A

bisoprolol

A beta-blocker or a calcium channel blocker is used first-line to prevent angina attacks

Amlodipine is not a rate-limiting calcium channel blocker so would not be used as first-line therapy.

47
Q

A 52-year-old male presents with central chest pain. On examination he has an mitral regurgitation murmur. An ECG shows ST elevation in leads V1 to V6. There is no ST elevation in leads II, III and aVF.

A

anterior MI

48
Q

A 52-year-old male presents with tearing central chest pain. On examination he has an aortic regurgitation murmur. An ECG shows ST elevation in leads II, III and aVF. most likely diagnosis?

A

proximal aortic dissection

49
Q

Synchronised DC cardioversion takes place as per the tachyarrhythmia advanced life support algorithm.

Which part of the QRS complex is used for synchronisation?

A

R wave

50
Q

A 78-year-old man presents to the emergency department with left-sided hemiparesis and a reduced conscious level. He has a history of hypertension, atrial fibrillation, and diabetes. He takes ramipril, amlodipine, dabigatran, and metformin.

On examination, his Glasgow coma scale is 13/15 and he has a facial weakness, slurred speech, and reduced power of his left upper and lower limb. An urgent CT scan is performed and shows a large right intra-cerebral hemorrhage.

What would be the single best initial treatment option?

A

Bleeding on dabigatran? Can use idarucizumab to reverse

51
Q

A 65-year-old man is on the ward recovering from an ST-elevated myocardial infarction (STEMI).

Five days into his recovery, he reports sudden onset shortness of breath, particularly when lying flat. He also reports developing a wheezing cough.

Examination reveals distended neck veins and an audible pan-systolic murmur.

Based on the information provided, which of the following pathologies is the most likely explanation for this patient’s current presentation?

A

VSD - pan systolic murmur

52
Q

An 85 year old man has attended surgery to discuss an ambulatory blood pressure monitoring reading of 142/84 mmHg. He has no past medical history of coronary heart disease, renal disease or diabetes, and his only regular medication is lansoprazole. His 10-year cardiovascular risk score was recently calculated to be 8%. Management should include follow up with which one of the following?

A

lifestyle advice
An ambulatory blood pressure reading of greater than or equal to 135/85 mmHg confirms a diagnosis of stage 1 hypertension. However, the National Institute for Clinical Excellence (NICE) suggest that antihypertensive treatment should be offered only if the person is:
aged less than 80 years with stage 1 hypertension with one or more of, target organ damage, established cardiovascular disease, renal disease, diabetes, and/or a 10 year cardiovascular risk of 10% or more.

53
Q

how should adenosine be administered?

A

Insert a 16G cannula in her right antecubital vein

54
Q

S3 is seen in LVF

A
55
Q

ECG shows the presence of long straight lines preceding QRS complexes

most likely cause?

A

pacemaker

56
Q

Amiodarone can cause a grey skin appearance

A
57
Q

A 67-year-old male presents to the emergency department with sudden onset chest pain. The pain is located in his central chest, and started an hour ago. The pain was maximal at onset, and is not exacerbated with deep breaths. He describes it as the most intense pain he’s ever experienced. He has not had any similar episodes previously.

He has a past medical history of hypertension (for which he takes ramipril and bendroflumethiazide). He has a 15-pack-year smoking history.

On examination he appears drowsy. He has left-sided ptosis and miosis of his left pupil.

What is the most likely cause of this presentation?

A

aortic dissection - can present with neurological complaints

58
Q

A 75-year-old woman presents with an 8-hour history of palpitations and shortness of breath. This is the first time she has had palpitations. On examination, she has peripheral oedema and a displaced apex beat. Her pulse is irregularly irregular at a rate of 110 bpm, her respiratory rate is 25 /min, her blood pressure is 105/67 mmHg, and a third heart sound and a pansystolic murmur are heard on auscultation. An ECG demonstrates absent p waves.

She has a past medical history of ischaemic heart disease and heart failure and takes ramipril, bisoprolol and furosemide.

What is the most appropriate management option?

A

amiodarone!!!

Use rhythm control to treat AF if there is coexistent heart failure, first onset AF or an obvious reversible cause

59
Q
A
60
Q
A
61
Q

ECG findings in pericarditis?

A

Concave ST elevation and PR depression

61
Q

INR 5.0-8.0 (no bleeding) - withhold 1 or 2 doses of warfarin, reduce subsequent maintenance dose

  • no bleeding so you dont need Vitamin K
A
62
Q

A 68-year-old man presents to the emergency department with a 2-hour history of tearing chest pain. His observations show:

Respiratory rate of 33 breaths/min
Pulse of 153 beats/min
Temperature of 37.2ºC
Blood pressure of 88/68mmHg
Oxygen saturations of 95% on room air

Auscultation of the heart identifies a diastolic murmur, heard loudest over the 2nd intercostal space, right sternal border. A chest x-ray is ordered, which shows a widened mediastinum.

Which of the following investigations would be most suitable for this patient?

A

TOE

as patient is unstable for CT angio

63
Q

A 28-year-old who is 10 weeks pregnant is noted to be hypertensive on her booking visit. Blood show a potassium of 2.9 mmol/l. Clinical examination is unremarkable

most likely diagnosis?

A

primary hyperaldosteronism

At 10 weeks gestation pregnancy-induced hypertension is not a possibility. The booking visit may represent the first time this patient has had her blood pressure checked, revealing an long-standing disorder. The low potassium points to a diagnosis of primary hyperaldosteronism (of which Conn’s syndrome is a subtype)

64
Q

An ECG shows broad complex tachycardia, consistent with a new-onset left bundle branch block compared to a previous ECG.

Given this information, what is the most appropriate next investigation?

A

troponin!! -> rule out ACS

65
Q

what medication should be avoided in patients with HOCM?

A

Ace inhibitors

66
Q

Poorly controlled hypertension, already taking an ACE inhibitor, calcium channel blocker and a standard-dose thiazide diuretic

what do you add?

A

alpha blocker or beta blocker

eg doxazosin

67
Q

constrictive peridcarditis JVP sign?

A

JVP increasing with inspiration is known as Kussmaul’s sign

68
Q

acute pericarditis management?

A

Ibuprofen and colchicine

69
Q

A 62-year-old woman presents to the emergency department complaining of worsening somnolence and confusion. She denies any shortness of breath or any other symptoms.

She has a past medical history of morbid obesity, chronic obstructive pulmonary disease, hypertension, heart failure and diabetes mellitus.

On examination, lower extremity oedema is noticeable. The doctor orders some blood tests which show the hyponatremia, name a drug that may have caused this?

A

furosemide/loop diuretics

70
Q

A 14-year-old child attends the emergency department following a bout of haemoptysis. He has been a well child with no regular contact with his doctor other than for vaccinations, which are up to date.

The only finding of note in his past history is the documentation of a harsh, blowing, pansystolic murmur noted during routine examination as a baby. There has never been any follow up of this and today on examination you cannot hear this murmur.

Additionally on examination you note loss of the nail fold angle and a blue tinge to the lips.

Given the likely diagnosis, what investigation findings would you expect?

A

ECG showing right ventricular hypertrophy

eissenmengers syndrome

71
Q

HOCM may occasionally be associated with a bisferiens pulse/ double pulse

A
72
Q

A 57-year-old male is referred to the cardiology clinic with mild dyspnoea. He has a background medical history of hypertension, hypercholesterolemia, type two diabetes mellitus, and he has recently been diagnosed with a metastatic carcinoid tumour.

On examination, he has a harsh mid-ejection systolic murmur that is loudest on inspiration.

What is the most likely diagnosis?

A

pulmonary stenosis

73
Q

A 65-year-old male presents with left sided hemiparesis, and decreased level of consciousness. On examination he has a blood pressure of 145/75 mmHg and pulse 110 beats per minute (regular). On auscultation he has crepitations to the mid zones and mild ankle oedema. He has a past medical history of a myocardial infarction 4 months previously. An ECG confirms persistent ST elevation in leads V1-V4.

What is the most likely cause of the stroke?

A

Left ventricular thromboembolism
60%

Persistent ST elevation after previous MI, is very suggestive of a left ventricle aneurysm. Blood stagnates around a left ventricle aneurysm, thereby promoting platelet adherence and thrombus formation

74
Q

A 75-year-old woman presents with fatigue, dyspnoea and ankle swelling on a background of hypertension, osteoarthritis, gout and type 2 diabetes mellitus. On examination, her cheeks appear erythematous. She has an irregularly irregular pulse and a raised JVP. There is peripheral oedema up to the knees bilaterally. On auscultation, the 1st heart sound is loud, and there is an added low, rumbling diastolic murmur.

Which is the following is the most likely cause of this woman’s presentation?

A

Rheumatic fever is the most common cause of mitral stenosis

75
Q

Which one of the following electrolyte disturbances could lead to long QT syndrome?

A

hypokalemia